Question on the limit of a sequence if the sum of the sequence converges [duplicate]












0












$begingroup$



This question already has an answer here:




  • If a series converges, then the sequence of terms converges to $0$.

    3 answers




Suppose I have a sequence ${a_i}_{i=1}^{infty}$. I know that $a_i geq 0$ for all $i$ and $sum_{i=1}^{infty}a_i < infty$. Can I say that



$$ lim_{i to infty} a_i = 0$$



Intuitively, I think this is true. If the limit tends to a positive number, the sum would explode as well. However, I'm having a hard time proving it formally. Can I get a hint?










share|cite|improve this question











$endgroup$



marked as duplicate by Martin R, Cesareo, Arnaud D., José Carlos Santos sequences-and-series
Users with the  sequences-and-series badge can single-handedly close sequences-and-series questions as duplicates and reopen them as needed.

StackExchange.ready(function() {
if (StackExchange.options.isMobile) return;

$('.dupe-hammer-message-hover:not(.hover-bound)').each(function() {
var $hover = $(this).addClass('hover-bound'),
$msg = $hover.siblings('.dupe-hammer-message');

$hover.hover(
function() {
$hover.showInfoMessage('', {
messageElement: $msg.clone().show(),
transient: false,
position: { my: 'bottom left', at: 'top center', offsetTop: -7 },
dismissable: false,
relativeToBody: true
});
},
function() {
StackExchange.helpers.removeMessages();
}
);
});
});
Jan 17 at 12:34


This question has been asked before and already has an answer. If those answers do not fully address your question, please ask a new question.


















  • $begingroup$
    Btw, this is the first topic in en.wikipedia.org/wiki/Convergence_tests#List_of_tests .
    $endgroup$
    – Martin R
    Jan 17 at 9:11


















0












$begingroup$



This question already has an answer here:




  • If a series converges, then the sequence of terms converges to $0$.

    3 answers




Suppose I have a sequence ${a_i}_{i=1}^{infty}$. I know that $a_i geq 0$ for all $i$ and $sum_{i=1}^{infty}a_i < infty$. Can I say that



$$ lim_{i to infty} a_i = 0$$



Intuitively, I think this is true. If the limit tends to a positive number, the sum would explode as well. However, I'm having a hard time proving it formally. Can I get a hint?










share|cite|improve this question











$endgroup$



marked as duplicate by Martin R, Cesareo, Arnaud D., José Carlos Santos sequences-and-series
Users with the  sequences-and-series badge can single-handedly close sequences-and-series questions as duplicates and reopen them as needed.

StackExchange.ready(function() {
if (StackExchange.options.isMobile) return;

$('.dupe-hammer-message-hover:not(.hover-bound)').each(function() {
var $hover = $(this).addClass('hover-bound'),
$msg = $hover.siblings('.dupe-hammer-message');

$hover.hover(
function() {
$hover.showInfoMessage('', {
messageElement: $msg.clone().show(),
transient: false,
position: { my: 'bottom left', at: 'top center', offsetTop: -7 },
dismissable: false,
relativeToBody: true
});
},
function() {
StackExchange.helpers.removeMessages();
}
);
});
});
Jan 17 at 12:34


This question has been asked before and already has an answer. If those answers do not fully address your question, please ask a new question.


















  • $begingroup$
    Btw, this is the first topic in en.wikipedia.org/wiki/Convergence_tests#List_of_tests .
    $endgroup$
    – Martin R
    Jan 17 at 9:11
















0












0








0





$begingroup$



This question already has an answer here:




  • If a series converges, then the sequence of terms converges to $0$.

    3 answers




Suppose I have a sequence ${a_i}_{i=1}^{infty}$. I know that $a_i geq 0$ for all $i$ and $sum_{i=1}^{infty}a_i < infty$. Can I say that



$$ lim_{i to infty} a_i = 0$$



Intuitively, I think this is true. If the limit tends to a positive number, the sum would explode as well. However, I'm having a hard time proving it formally. Can I get a hint?










share|cite|improve this question











$endgroup$





This question already has an answer here:




  • If a series converges, then the sequence of terms converges to $0$.

    3 answers




Suppose I have a sequence ${a_i}_{i=1}^{infty}$. I know that $a_i geq 0$ for all $i$ and $sum_{i=1}^{infty}a_i < infty$. Can I say that



$$ lim_{i to infty} a_i = 0$$



Intuitively, I think this is true. If the limit tends to a positive number, the sum would explode as well. However, I'm having a hard time proving it formally. Can I get a hint?





This question already has an answer here:




  • If a series converges, then the sequence of terms converges to $0$.

    3 answers








sequences-and-series limits summation






share|cite|improve this question















share|cite|improve this question













share|cite|improve this question




share|cite|improve this question








edited Jan 17 at 6:59







user1691278

















asked Jan 16 at 23:24









user1691278user1691278

48739




48739




marked as duplicate by Martin R, Cesareo, Arnaud D., José Carlos Santos sequences-and-series
Users with the  sequences-and-series badge can single-handedly close sequences-and-series questions as duplicates and reopen them as needed.

StackExchange.ready(function() {
if (StackExchange.options.isMobile) return;

$('.dupe-hammer-message-hover:not(.hover-bound)').each(function() {
var $hover = $(this).addClass('hover-bound'),
$msg = $hover.siblings('.dupe-hammer-message');

$hover.hover(
function() {
$hover.showInfoMessage('', {
messageElement: $msg.clone().show(),
transient: false,
position: { my: 'bottom left', at: 'top center', offsetTop: -7 },
dismissable: false,
relativeToBody: true
});
},
function() {
StackExchange.helpers.removeMessages();
}
);
});
});
Jan 17 at 12:34


This question has been asked before and already has an answer. If those answers do not fully address your question, please ask a new question.









marked as duplicate by Martin R, Cesareo, Arnaud D., José Carlos Santos sequences-and-series
Users with the  sequences-and-series badge can single-handedly close sequences-and-series questions as duplicates and reopen them as needed.

StackExchange.ready(function() {
if (StackExchange.options.isMobile) return;

$('.dupe-hammer-message-hover:not(.hover-bound)').each(function() {
var $hover = $(this).addClass('hover-bound'),
$msg = $hover.siblings('.dupe-hammer-message');

$hover.hover(
function() {
$hover.showInfoMessage('', {
messageElement: $msg.clone().show(),
transient: false,
position: { my: 'bottom left', at: 'top center', offsetTop: -7 },
dismissable: false,
relativeToBody: true
});
},
function() {
StackExchange.helpers.removeMessages();
}
);
});
});
Jan 17 at 12:34


This question has been asked before and already has an answer. If those answers do not fully address your question, please ask a new question.














  • $begingroup$
    Btw, this is the first topic in en.wikipedia.org/wiki/Convergence_tests#List_of_tests .
    $endgroup$
    – Martin R
    Jan 17 at 9:11




















  • $begingroup$
    Btw, this is the first topic in en.wikipedia.org/wiki/Convergence_tests#List_of_tests .
    $endgroup$
    – Martin R
    Jan 17 at 9:11


















$begingroup$
Btw, this is the first topic in en.wikipedia.org/wiki/Convergence_tests#List_of_tests .
$endgroup$
– Martin R
Jan 17 at 9:11






$begingroup$
Btw, this is the first topic in en.wikipedia.org/wiki/Convergence_tests#List_of_tests .
$endgroup$
– Martin R
Jan 17 at 9:11












3 Answers
3






active

oldest

votes


















5












$begingroup$

We have
$$sum_{i=0}^n a_ito S ,qquad sum_{i=0}^{n-1} a_ito S$$
as $ntoinfty$, so
$$a_n=Bigl(sum_{i=0}^n a_iBigr)-Bigl(sum_{i=0}^{n-1} a_iBigr)to S-S=0 .$$






share|cite|improve this answer









$endgroup$













  • $begingroup$
    Great proof. Is $a_i > 0$ even necessary?
    $endgroup$
    – user1691278
    Jan 16 at 23:30












  • $begingroup$
    No.$!,!,!,$
    $endgroup$
    – David
    Jan 16 at 23:31



















0












$begingroup$

Informal outline for a more formal proof:



Let $s_n$ be the sum of the first $n$ terms and let the limit of the series be $m$.



Convergence means that for as small an $epsilon>0$ as we like and large enough $N$, $n>N$ guarantees $|m-s_n|<epsilon$.



But this goes awry if ${a_n}$ doesn't converge to $0$, because when $2epsilon<|a_{n+1}|$ it's not possible to have both $|m-s_n|<epsilon$ and $|m-s_{n+1}|<epsilon$.



(And use the definition of ${a_n}$ not converging to show that this situation must arise.)






share|cite|improve this answer











$endgroup$





















    0












    $begingroup$

    I'm not quite sure if the sequence starts at index $0$ or $1$ as your question uses $1$ as the sequence start but the summation starts at $0$. As it's not important, I will assume it's $0$. Note that



    $$sum_{i , = , 0}^{infty} a_i lt infty tag{1}label{eq1}$$



    usually means it converges to a limit. However, consider the somewhat looser restriction of there just simply existing a real supremum, call it $S$, so that



    $$sum_{i , = , 0}^{n} a_i le S ; forall ; n ge 0 tag{2}label{eq2}$$



    As for trying to show the limit of $a_i$ is $0$, I will just let the "$- ; 0$" part be implied. Also, it's given that $a_i ge 0$, so the partial sums are non-decreasing, and there are no non-negative values so I don't need to use absolute values in the following. For proving that



    $$lim_{i , to , infty} a_i = 0 tag{3}label{eq3}$$



    consider that for any $epsilon gt 0$ there must be a finite number of integers for which $a_n ge epsilon$ as, otherwise, the sum of them would be unbounded. Thus, there must be either none or a maximum index, but in either choose an integer $n_0$ to be greater than any such value and, thus, $a_n lt epsilon$ for all $n gt n_0$, which confirms that eqref{eq3} is true. I won't show it here, but the infinite sum has a limit as well of $S$.



    Note that assuming just a supremum for the sums without assuming that $a_i ge 0$ won't suffice to show the limit as, for example, $a_i = left(-1right)^i$ would cause the partial sums to alternate between $1$ and $0$, thus allowing a supremum of $1$, but with $a_i$ not converging to $0$.



    Note that requiring the sum of $a_i$ to have a finite limit means that, as David indicated in his answer's comment, also requiring the $a_i$ to be non-negative is unnecessary. However, if the requirement of eqref{eq1} includes the possibility of the limit being $-infty$, then some appropriate restrictions on $a_i$ would be required.






    share|cite|improve this answer











    $endgroup$




















      3 Answers
      3






      active

      oldest

      votes








      3 Answers
      3






      active

      oldest

      votes









      active

      oldest

      votes






      active

      oldest

      votes









      5












      $begingroup$

      We have
      $$sum_{i=0}^n a_ito S ,qquad sum_{i=0}^{n-1} a_ito S$$
      as $ntoinfty$, so
      $$a_n=Bigl(sum_{i=0}^n a_iBigr)-Bigl(sum_{i=0}^{n-1} a_iBigr)to S-S=0 .$$






      share|cite|improve this answer









      $endgroup$













      • $begingroup$
        Great proof. Is $a_i > 0$ even necessary?
        $endgroup$
        – user1691278
        Jan 16 at 23:30












      • $begingroup$
        No.$!,!,!,$
        $endgroup$
        – David
        Jan 16 at 23:31
















      5












      $begingroup$

      We have
      $$sum_{i=0}^n a_ito S ,qquad sum_{i=0}^{n-1} a_ito S$$
      as $ntoinfty$, so
      $$a_n=Bigl(sum_{i=0}^n a_iBigr)-Bigl(sum_{i=0}^{n-1} a_iBigr)to S-S=0 .$$






      share|cite|improve this answer









      $endgroup$













      • $begingroup$
        Great proof. Is $a_i > 0$ even necessary?
        $endgroup$
        – user1691278
        Jan 16 at 23:30












      • $begingroup$
        No.$!,!,!,$
        $endgroup$
        – David
        Jan 16 at 23:31














      5












      5








      5





      $begingroup$

      We have
      $$sum_{i=0}^n a_ito S ,qquad sum_{i=0}^{n-1} a_ito S$$
      as $ntoinfty$, so
      $$a_n=Bigl(sum_{i=0}^n a_iBigr)-Bigl(sum_{i=0}^{n-1} a_iBigr)to S-S=0 .$$






      share|cite|improve this answer









      $endgroup$



      We have
      $$sum_{i=0}^n a_ito S ,qquad sum_{i=0}^{n-1} a_ito S$$
      as $ntoinfty$, so
      $$a_n=Bigl(sum_{i=0}^n a_iBigr)-Bigl(sum_{i=0}^{n-1} a_iBigr)to S-S=0 .$$







      share|cite|improve this answer












      share|cite|improve this answer



      share|cite|improve this answer










      answered Jan 16 at 23:29









      DavidDavid

      68.6k665127




      68.6k665127












      • $begingroup$
        Great proof. Is $a_i > 0$ even necessary?
        $endgroup$
        – user1691278
        Jan 16 at 23:30












      • $begingroup$
        No.$!,!,!,$
        $endgroup$
        – David
        Jan 16 at 23:31


















      • $begingroup$
        Great proof. Is $a_i > 0$ even necessary?
        $endgroup$
        – user1691278
        Jan 16 at 23:30












      • $begingroup$
        No.$!,!,!,$
        $endgroup$
        – David
        Jan 16 at 23:31
















      $begingroup$
      Great proof. Is $a_i > 0$ even necessary?
      $endgroup$
      – user1691278
      Jan 16 at 23:30






      $begingroup$
      Great proof. Is $a_i > 0$ even necessary?
      $endgroup$
      – user1691278
      Jan 16 at 23:30














      $begingroup$
      No.$!,!,!,$
      $endgroup$
      – David
      Jan 16 at 23:31




      $begingroup$
      No.$!,!,!,$
      $endgroup$
      – David
      Jan 16 at 23:31











      0












      $begingroup$

      Informal outline for a more formal proof:



      Let $s_n$ be the sum of the first $n$ terms and let the limit of the series be $m$.



      Convergence means that for as small an $epsilon>0$ as we like and large enough $N$, $n>N$ guarantees $|m-s_n|<epsilon$.



      But this goes awry if ${a_n}$ doesn't converge to $0$, because when $2epsilon<|a_{n+1}|$ it's not possible to have both $|m-s_n|<epsilon$ and $|m-s_{n+1}|<epsilon$.



      (And use the definition of ${a_n}$ not converging to show that this situation must arise.)






      share|cite|improve this answer











      $endgroup$


















        0












        $begingroup$

        Informal outline for a more formal proof:



        Let $s_n$ be the sum of the first $n$ terms and let the limit of the series be $m$.



        Convergence means that for as small an $epsilon>0$ as we like and large enough $N$, $n>N$ guarantees $|m-s_n|<epsilon$.



        But this goes awry if ${a_n}$ doesn't converge to $0$, because when $2epsilon<|a_{n+1}|$ it's not possible to have both $|m-s_n|<epsilon$ and $|m-s_{n+1}|<epsilon$.



        (And use the definition of ${a_n}$ not converging to show that this situation must arise.)






        share|cite|improve this answer











        $endgroup$
















          0












          0








          0





          $begingroup$

          Informal outline for a more formal proof:



          Let $s_n$ be the sum of the first $n$ terms and let the limit of the series be $m$.



          Convergence means that for as small an $epsilon>0$ as we like and large enough $N$, $n>N$ guarantees $|m-s_n|<epsilon$.



          But this goes awry if ${a_n}$ doesn't converge to $0$, because when $2epsilon<|a_{n+1}|$ it's not possible to have both $|m-s_n|<epsilon$ and $|m-s_{n+1}|<epsilon$.



          (And use the definition of ${a_n}$ not converging to show that this situation must arise.)






          share|cite|improve this answer











          $endgroup$



          Informal outline for a more formal proof:



          Let $s_n$ be the sum of the first $n$ terms and let the limit of the series be $m$.



          Convergence means that for as small an $epsilon>0$ as we like and large enough $N$, $n>N$ guarantees $|m-s_n|<epsilon$.



          But this goes awry if ${a_n}$ doesn't converge to $0$, because when $2epsilon<|a_{n+1}|$ it's not possible to have both $|m-s_n|<epsilon$ and $|m-s_{n+1}|<epsilon$.



          (And use the definition of ${a_n}$ not converging to show that this situation must arise.)







          share|cite|improve this answer














          share|cite|improve this answer



          share|cite|improve this answer








          edited Jan 17 at 1:09

























          answered Jan 17 at 0:43









          timtfjtimtfj

          2,198420




          2,198420























              0












              $begingroup$

              I'm not quite sure if the sequence starts at index $0$ or $1$ as your question uses $1$ as the sequence start but the summation starts at $0$. As it's not important, I will assume it's $0$. Note that



              $$sum_{i , = , 0}^{infty} a_i lt infty tag{1}label{eq1}$$



              usually means it converges to a limit. However, consider the somewhat looser restriction of there just simply existing a real supremum, call it $S$, so that



              $$sum_{i , = , 0}^{n} a_i le S ; forall ; n ge 0 tag{2}label{eq2}$$



              As for trying to show the limit of $a_i$ is $0$, I will just let the "$- ; 0$" part be implied. Also, it's given that $a_i ge 0$, so the partial sums are non-decreasing, and there are no non-negative values so I don't need to use absolute values in the following. For proving that



              $$lim_{i , to , infty} a_i = 0 tag{3}label{eq3}$$



              consider that for any $epsilon gt 0$ there must be a finite number of integers for which $a_n ge epsilon$ as, otherwise, the sum of them would be unbounded. Thus, there must be either none or a maximum index, but in either choose an integer $n_0$ to be greater than any such value and, thus, $a_n lt epsilon$ for all $n gt n_0$, which confirms that eqref{eq3} is true. I won't show it here, but the infinite sum has a limit as well of $S$.



              Note that assuming just a supremum for the sums without assuming that $a_i ge 0$ won't suffice to show the limit as, for example, $a_i = left(-1right)^i$ would cause the partial sums to alternate between $1$ and $0$, thus allowing a supremum of $1$, but with $a_i$ not converging to $0$.



              Note that requiring the sum of $a_i$ to have a finite limit means that, as David indicated in his answer's comment, also requiring the $a_i$ to be non-negative is unnecessary. However, if the requirement of eqref{eq1} includes the possibility of the limit being $-infty$, then some appropriate restrictions on $a_i$ would be required.






              share|cite|improve this answer











              $endgroup$


















                0












                $begingroup$

                I'm not quite sure if the sequence starts at index $0$ or $1$ as your question uses $1$ as the sequence start but the summation starts at $0$. As it's not important, I will assume it's $0$. Note that



                $$sum_{i , = , 0}^{infty} a_i lt infty tag{1}label{eq1}$$



                usually means it converges to a limit. However, consider the somewhat looser restriction of there just simply existing a real supremum, call it $S$, so that



                $$sum_{i , = , 0}^{n} a_i le S ; forall ; n ge 0 tag{2}label{eq2}$$



                As for trying to show the limit of $a_i$ is $0$, I will just let the "$- ; 0$" part be implied. Also, it's given that $a_i ge 0$, so the partial sums are non-decreasing, and there are no non-negative values so I don't need to use absolute values in the following. For proving that



                $$lim_{i , to , infty} a_i = 0 tag{3}label{eq3}$$



                consider that for any $epsilon gt 0$ there must be a finite number of integers for which $a_n ge epsilon$ as, otherwise, the sum of them would be unbounded. Thus, there must be either none or a maximum index, but in either choose an integer $n_0$ to be greater than any such value and, thus, $a_n lt epsilon$ for all $n gt n_0$, which confirms that eqref{eq3} is true. I won't show it here, but the infinite sum has a limit as well of $S$.



                Note that assuming just a supremum for the sums without assuming that $a_i ge 0$ won't suffice to show the limit as, for example, $a_i = left(-1right)^i$ would cause the partial sums to alternate between $1$ and $0$, thus allowing a supremum of $1$, but with $a_i$ not converging to $0$.



                Note that requiring the sum of $a_i$ to have a finite limit means that, as David indicated in his answer's comment, also requiring the $a_i$ to be non-negative is unnecessary. However, if the requirement of eqref{eq1} includes the possibility of the limit being $-infty$, then some appropriate restrictions on $a_i$ would be required.






                share|cite|improve this answer











                $endgroup$
















                  0












                  0








                  0





                  $begingroup$

                  I'm not quite sure if the sequence starts at index $0$ or $1$ as your question uses $1$ as the sequence start but the summation starts at $0$. As it's not important, I will assume it's $0$. Note that



                  $$sum_{i , = , 0}^{infty} a_i lt infty tag{1}label{eq1}$$



                  usually means it converges to a limit. However, consider the somewhat looser restriction of there just simply existing a real supremum, call it $S$, so that



                  $$sum_{i , = , 0}^{n} a_i le S ; forall ; n ge 0 tag{2}label{eq2}$$



                  As for trying to show the limit of $a_i$ is $0$, I will just let the "$- ; 0$" part be implied. Also, it's given that $a_i ge 0$, so the partial sums are non-decreasing, and there are no non-negative values so I don't need to use absolute values in the following. For proving that



                  $$lim_{i , to , infty} a_i = 0 tag{3}label{eq3}$$



                  consider that for any $epsilon gt 0$ there must be a finite number of integers for which $a_n ge epsilon$ as, otherwise, the sum of them would be unbounded. Thus, there must be either none or a maximum index, but in either choose an integer $n_0$ to be greater than any such value and, thus, $a_n lt epsilon$ for all $n gt n_0$, which confirms that eqref{eq3} is true. I won't show it here, but the infinite sum has a limit as well of $S$.



                  Note that assuming just a supremum for the sums without assuming that $a_i ge 0$ won't suffice to show the limit as, for example, $a_i = left(-1right)^i$ would cause the partial sums to alternate between $1$ and $0$, thus allowing a supremum of $1$, but with $a_i$ not converging to $0$.



                  Note that requiring the sum of $a_i$ to have a finite limit means that, as David indicated in his answer's comment, also requiring the $a_i$ to be non-negative is unnecessary. However, if the requirement of eqref{eq1} includes the possibility of the limit being $-infty$, then some appropriate restrictions on $a_i$ would be required.






                  share|cite|improve this answer











                  $endgroup$



                  I'm not quite sure if the sequence starts at index $0$ or $1$ as your question uses $1$ as the sequence start but the summation starts at $0$. As it's not important, I will assume it's $0$. Note that



                  $$sum_{i , = , 0}^{infty} a_i lt infty tag{1}label{eq1}$$



                  usually means it converges to a limit. However, consider the somewhat looser restriction of there just simply existing a real supremum, call it $S$, so that



                  $$sum_{i , = , 0}^{n} a_i le S ; forall ; n ge 0 tag{2}label{eq2}$$



                  As for trying to show the limit of $a_i$ is $0$, I will just let the "$- ; 0$" part be implied. Also, it's given that $a_i ge 0$, so the partial sums are non-decreasing, and there are no non-negative values so I don't need to use absolute values in the following. For proving that



                  $$lim_{i , to , infty} a_i = 0 tag{3}label{eq3}$$



                  consider that for any $epsilon gt 0$ there must be a finite number of integers for which $a_n ge epsilon$ as, otherwise, the sum of them would be unbounded. Thus, there must be either none or a maximum index, but in either choose an integer $n_0$ to be greater than any such value and, thus, $a_n lt epsilon$ for all $n gt n_0$, which confirms that eqref{eq3} is true. I won't show it here, but the infinite sum has a limit as well of $S$.



                  Note that assuming just a supremum for the sums without assuming that $a_i ge 0$ won't suffice to show the limit as, for example, $a_i = left(-1right)^i$ would cause the partial sums to alternate between $1$ and $0$, thus allowing a supremum of $1$, but with $a_i$ not converging to $0$.



                  Note that requiring the sum of $a_i$ to have a finite limit means that, as David indicated in his answer's comment, also requiring the $a_i$ to be non-negative is unnecessary. However, if the requirement of eqref{eq1} includes the possibility of the limit being $-infty$, then some appropriate restrictions on $a_i$ would be required.







                  share|cite|improve this answer














                  share|cite|improve this answer



                  share|cite|improve this answer








                  edited Jan 17 at 8:04

























                  answered Jan 17 at 0:23









                  John OmielanJohn Omielan

                  2,561212




                  2,561212















                      Popular posts from this blog

                      Mario Kart Wii

                      What does “Dominus providebit” mean?

                      Antonio Litta Visconti Arese